How do I solve a circuit with multiple nodes and loops?

Click For Summary
The discussion focuses on solving a circuit with multiple nodes and loops, where the user is attempting to set up equations based on Kirchhoff's laws. They have identified four nodes and two loops but are uncertain about their equations and whether they have accounted for all necessary nodes and loops. The user suggests needing seven equations for seven unknowns but is unsure about the setup. A response indicates that at the junctions, the currents must balance, providing an additional equation to consider. Clarification on the correct number of equations and their formulation is sought to resolve the circuit analysis.
tylersmith7690
Messages
21
Reaction score
0

Homework Statement



In attatchment


Homework Equations



Not sure How to come up with the other equations.

The Attempt at a Solution



Assuming a node is an element connected to two or more elements.

1) Moving clockwise from V i1 to i2. Node one is between the negative side of the voltage source and first resistor.

2) Node 2 is from first resistor to second resistor on the right and 3rd resistor down the middle.

3) Node 3 is the top right hand corner between those two resistors.

4) node 4 is the entire wire running down from the far right resistor and middle resistor and to the voltage supply.

I also have two loops.
Loop A encompass i2 i7 and i6 and loop B encompasses i3 i4 i5 and the middle resistor.

I'm having trouble setting up the equations as all the references I've looked at have done the questions a little different. Mainly just using the junction method.
current entering = current out.

I have no experience on circuits so this is just what I make of the information I've read.

eq 1 ) using node 1. i1 - i2 = 0
eq 2) using node 2. i2 - i3 - i7 = 0
eq 3) using node 3. i3 - i4 = 0
eq 4) using node 4 i5 + i7 - i6 = 0
eq 5) loop a i2*R + i7*R - V = 0
eq 6) loop b i3*R + i4*R - i7*R = 0

this is where I'm stuck and I'm unsure about my above equations. Is there a outside loop around the entire circuit or is there a node i have missed. I assume i need 7 equations for the 7 unknowns.

Any help would be much appreciated. Kind regards.
 

Attachments

  • circuit.PNG
    circuit.PNG
    15.8 KB · Views: 431
Physics news on Phys.org
tylersmith7690 said:

Homework Statement



In attatchment


Homework Equations



Not sure How to come up with the other equations.

The Attempt at a Solution



Assuming a node is an element connected to two or more elements.

1) Moving clockwise from V i1 to i2. Node one is between the negative side of the voltage source and first resistor.

2) Node 2 is from first resistor to second resistor on the right and 3rd resistor down the middle.

3) Node 3 is the top right hand corner between those two resistors.

4) node 4 is the entire wire running down from the far right resistor and middle resistor and to the voltage supply.

I also have two loops.
Loop A encompass i2 i7 and i6 and loop B encompasses i3 i4 i5 and the middle resistor.

I'm having trouble setting up the equations as all the references I've looked at have done the questions a little different. Mainly just using the junction method.
current entering = current out.

I have no experience on circuits so this is just what I make of the information I've read.

eq 1 ) using node 1. i1 - i2 = 0
eq 2) using node 2. i2 - i3 - i7 = 0
eq 3) using node 3. i3 - i4 = 0
eq 4) using node 4 i5 + i7 - i6 = 0
eq 5) loop a i2*R + i7*R - V = 0
eq 6) loop b i3*R + i4*R - i7*R = 0

this is where I'm stuck and I'm unsure about my above equations. Is there a outside loop around the entire circuit or is there a node i have missed. I assume i need 7 equations for the 7 unknowns.

Any help would be much appreciated. Kind regards.

Basically, you just have three different currents, i1, i4 and i7. At the top t-junction you have i1 = i4 + i7, and this applies as well at the bottom t-junction (check this!). So that is one equation, and you need two more. Can you see what those other two equations must be?
 
Question: A clock's minute hand has length 4 and its hour hand has length 3. What is the distance between the tips at the moment when it is increasing most rapidly?(Putnam Exam Question) Answer: Making assumption that both the hands moves at constant angular velocities, the answer is ## \sqrt{7} .## But don't you think this assumption is somewhat doubtful and wrong?

Similar threads

Replies
10
Views
2K
Replies
4
Views
1K
  • · Replies 4 ·
Replies
4
Views
2K
Replies
8
Views
2K
  • · Replies 4 ·
Replies
4
Views
3K
  • · Replies 9 ·
Replies
9
Views
3K
  • · Replies 3 ·
Replies
3
Views
2K
Replies
15
Views
2K
  • · Replies 4 ·
Replies
4
Views
2K
Replies
22
Views
4K